LSAT and Law School Admissions Forum

Get expert LSAT preparation and law school admissions advice from PowerScore Test Preparation.

 bk1111
  • Posts: 103
  • Joined: Apr 22, 2017
|
#45521
Can someone tell me if I negated answer choice B correctly:

My negation: Some people who do not serve on a medical review panel do have the necessary knowledge and expertise to evaluate..." I think this does not weaken/destroy the conclusion because the stimulus limits the use of prepublication peer review for those in the public that are "ill equipped" to evaluate medical findings. Is this correct?

Similarly, can someone tell me if my negation for A is correct: "Even if medical research findings are not bought to peer review by medical journal, peer review will not occur"
 Malila Robinson
PowerScore Staff
  • PowerScore Staff
  • Posts: 296
  • Joined: Feb 01, 2018
|
#45728
Hi bk1111,
In B your negation would be better if you said that they 'may have the necessary knowledge...' if you look on page one there is an explanation for this. And your reasoning for B looks good.

Answer A might be easier to understand if you say something like: Without medical findings being brought to peer review by a medical journal, peer review may still occur.

Hope that helps,
Malila
 gen2871
  • Posts: 47
  • Joined: Jul 01, 2018
|
#48949
Hi Dear LSAT Masters:
Please check if my reasoning is correct.
The stimulus mentions:
"MEDICAL RESEARCH findINGS are customarily not made PUBLIC prior to their publication in a MEDICAL JOURNAL that has ... (had)... PEER REVIEW. "
P1: (time sequence) Medical research ------Peer Review ------- Medical Journals ----- Public gets access
Medical findings published in a medical journal :arrow: customarily passed peer review

P2: Prevent error and harm :arrow: Peer review is required (the only, indicator of necessary condition)

Conclusion: to protect :arrow: peer review ------ medical journal is necessary.

Assumption, Peer review can only happen under medical journal.

A. Not published unless peer review + Medical journal.
B. it contains extraneous information, hence incorrect.
C. Contrary to the stimulus because it was mentioned once published, the public will have access.
D. Was wondering why did is not a good answer, but LSAT legend's explanation was spot on. "Customarily" is what is always done vs. the answer choice states what should be done confused. hence incorrect.
E. political and professional pressures are also extraneous information.

Please advise. Thank you!
Last edited by gen2871 on Wed Aug 01, 2018 4:35 am, edited 3 times in total.
 gen2871
  • Posts: 47
  • Joined: Jul 01, 2018
|
#48987
I loaded the wrong question under this one. It doesnt let me delete it.:(
Last edited by gen2871 on Wed Aug 01, 2018 4:12 am, edited 1 time in total.
 Adam Tyson
PowerScore Staff
  • PowerScore Staff
  • Posts: 5153
  • Joined: Apr 14, 2011
|
#49046
Good catch on the subsequent review, blueballoon! Trying the negation technique on C, we would get "the public has access to medical journals...". That doesn't hurt the argument that pre-publication peer review is needed, because this access is AFTER that process. Good work!
 chian9010
  • Posts: 81
  • Joined: Jun 08, 2018
|
#50014
I have a question about answer choice D.

If we negate it, it will become = not all medical research findings are subjected to prepublication peer review. The negated statement actually weaken the argument as if not all medical research findings need to go through the peer review process, then there won't be any time delayed in publishing the research. However, the accuracy of the research is not confirmed so the medical journal won't be able to protect the public which weaken the conclusion.
 Who Ray
PowerScore Staff
  • PowerScore Staff
  • Posts: 27
  • Joined: Jul 31, 2018
|
#50434
Hey there Chian!

I do not think a negated AC D actually breaks the argument the way it should in a negation test, and I have a couple nitty gritty reasons why. One reason is that findings are "customarily" kept secret, not necessarily all the time. Therefore, the argument can accommodate a journals that does not peer review things without falling apart. Another reason is that the conclusion specifies that waiting for a journal that does peer review is the price we pay—not "waiting for a journal to publish is the price we pay." That qualifier leaves room for journals without peer review, because the stimulus is not actually talking about all journals in the first place, only those with peer review.

Cheers,
Who Ray
 whardy21
  • Posts: 48
  • Joined: Sep 30, 2018
|
#66422
I selected answer choice B as my answer. My prephrase was: If the conclusion is true, the correct answer must be that the public will be unable to the understand the medical research findings. My prephrase led me to answer choice B. If waiting until a medical journal is peer reviewed to protect the public, I assumed that if the medical research gets into the public hands they will be subjected to harm due to not properly understanding the research. Am I far off in my prephrase? Please let me know.
User avatar
 KelseyWoods
PowerScore Staff
  • PowerScore Staff
  • Posts: 1079
  • Joined: Jun 26, 2013
|
#66817
Hi Whardy!

There are already some good explanations about why answer choice (B) is incorrect and why answer choice (A) is correct on the first page of this thread so I'm going to focus on your more general concern about prephrasing--it can be a tricky skill to master but is so worth it for truly being successful on the LSAT!

When you're prephrasing an assumption question, you need to be a little careful about prephrasing super specifically because most arguments have multiple assumptions. Your prephrase was not a bad one--one assumption of this argument is that the general public is unable to evaluate medical claims on its own. But that's not the only assumption of this argument. And because you were tied to your original prephrase, it led you to a not so great answer choice. Answer choice (B) doesn't actually match your prephrase, it's only SIMILAR to your prephrase. But it goes too far: it's not necessary that ANYONE not on a medical review panel is unable to evaluate research findings--just that enough of the public is unable to do so that it could be potentially harmful. And that's what we need to be careful of...don't allow your prephrase to lead you to choosing an answer choice that reminds you of your prephrase but doesn't actually fit your prephrase.

Try to prephrase broadly enough to allow you to recognize answer choices that are still good even though they weren't what you had specifically thought of earlier. And always rely on techniques like the Assumption Negation Technique to check yourself and make sure you aren't falling for a substandard answer choice.

Hope this helps!

Best,
Kelsey
User avatar
 JoshuaDEL
  • Posts: 15
  • Joined: Apr 25, 2021
|
#87134
Hello, I'm having trouble with why A is the answer.

The text says in the last sentence, "Therefore, waiting until a medical journal has published the research findings that have passed peer review " I don't see where the argument makes assumptions. If it said "Therefore, waiting until a medical journal has published the research findings..." then, I would agree that the author made that assumption. But the author specifically mentions "that have passed peer review". If anything, author this tells me that the author is NOT making that assumption stated in choice A.

Get the most out of your LSAT Prep Plus subscription.

Analyze and track your performance with our Testing and Analytics Package.